Questions tagged [analytic-number-theory]

A beautiful blending of real/complex analysis with number theory. The study involves distribution of prime numbers and other problems and helps giving asymptotic estimates to these.

942 questions with no upvoted or accepted answers
Filter by
Sorted by
Tagged with
35 votes
0 answers
1k views

Is there a rigid analytic geometry proof of the functional equation for the Riemann zeta function?

The adèles $\mathbb A$ arise naturally when considering the Berkovich space $\mathcal M(\mathbb Z)$ of the integers. Namely, they are the stalk $\mathbb A = (j_\ast j^{-1} \mathcal O_\mathbb Z)_p$ ...
Tim Campion's user avatar
  • 61.5k
26 votes
0 answers
526 views

Elliptic analogue of primes of the form $x^2 + 1$

I have a project in mind for an undergraduate to investigate next quarter -- a curiosity really, but I'm surprised I can't find it in the literature. I do not want a detailed analysis here... but ...
Marty's user avatar
  • 13.1k
23 votes
0 answers
12k views

Philosophy behind Zhang's 2022 preprint on the Landau–Siegel zero

Now that a week has passed since Zhang posted his preprint Discrete mean estimates and the Landau–Siegel zero on the arXiv, I'm wondering if someone can give a high-level overview of his strategy. In ...
Stopple's user avatar
  • 10.8k
19 votes
0 answers
506 views

Checking Mertens and the like in less than linear time or less than $\sqrt{x}$ space

Say you want to check that $|\sum_{n\leq x} \mu(n)|\leq \sqrt{x}$ for all $x\leq X$. (I am actually interested in checking that $\sum_{n\leq x} \mu(n)/n|\leq c/\sqrt{x}$, where $c$ is a constant, and ...
H A Helfgott's user avatar
  • 19.4k
19 votes
0 answers
3k views

sums of digits of powers of integers

It is known (Senge and Straus, 1971, see also C.L.Stewart, 1980) that for every natural $a $, not a power of 10, and every natural $s$, there are only finitely many $k$ such that the sum of decimal ...
user avatar
18 votes
0 answers
575 views

Consecutive integers of the form $2^a 3^b 5^c$

Let $\mathcal{N}$ denote the set of all products of (powers of) $2,3$ and $5$: $$ \mathcal{N} = \{ 2^a 3^b 5^c \ : \ a,b,c \geq 0 \} \subset \mathbb{N}.$$ We use the elements of $\mathcal{N}$ to ...
Jakub Konieczny's user avatar
18 votes
0 answers
711 views

Infinite extensions such that every elliptic curve has finite rank

The comments to this answer seem to make the following claim. Claim. Let $K$ be the maximal abelian extension of $\mathbf Q$ that is unramified away from $p$ (more generally, away from a finite set $S$...
R. van Dobben de Bruyn's user avatar
16 votes
0 answers
346 views

Transcendence of sum of reciprocals of factorials

For $A \subseteq \mathbb{N}$, define $\displaystyle x_A = \sum_{n \in A} \frac{1}{n!}$. It is easy to see that for every infinite $A$, $x_A$ is irrational. Question: Is there an infinite $A \subseteq \...
Sam's user avatar
  • 161
15 votes
0 answers
337 views

Do primes of the form $4k+1$ ever lead the greatest prime factor race?

Analogous to Chebyshev's race between primes, I examined the race between primes in the greatest prime factors, GPF, of natural numbers. Similar to the regular prime race, in the GPF race, the ...
Nilotpal Kanti Sinha's user avatar
14 votes
0 answers
292 views

An 'onion-structure' for roots of a series associated to prime numbers?

The series $$\sum_{n=1}^\infty\frac{z^{p_n-n}}{n!}$$ associated to the sequence $p_1=2,p_2=3,p_3=5,p_4=7,p_5=11,\ldots$ of prime numbers defines a holomorphic function in the open disc of radius $e$. ...
Roland Bacher's user avatar
14 votes
0 answers
402 views

Is every prime $q$ of the form $x^2 + py^2$ for some prime $p<q$?

For every odd prime $q \geq 3$, does there exist a prime $p < q$ and integers $x,y$ such that $$\displaystyle x^2 + py^2 = q?$$ One can easily show that all primes $q \not \equiv -1 \pmod{3}$ can ...
Stanley Yao Xiao's user avatar
14 votes
0 answers
792 views

Growth of residues of $1/\zeta(s)$: conjectures?

Let $\rho$ range over the non-trivial zeroes of the Riemann zeta function. Let $$M(T) = \max_{|\Im \rho|\leq T} \left|\mathrm{Res}_{s=\rho} \frac{1}{\zeta(s)}\right| = \max_{|\Im \rho|\leq T} \frac{1}...
H A Helfgott's user avatar
  • 19.4k
14 votes
0 answers
557 views

Moments of derivatives of $L$-functions

I'd like to know why it is important to know the moments of the derivatives of $L$-functions. The moments of $L$-functions are related to the Lindelöf Hypothesis, but what about the moments of the ...
Anna's user avatar
  • 241
14 votes
0 answers
345 views

Quasiperiodic continued fractions

Is anything known about continued fractions in which the sequence of integers is quasiperiodic? Quasiperiodic is meant here in the sense of 1D quasicrystals. For example, draw an irrationally-sloped ...
F Flicker's user avatar
  • 141
14 votes
0 answers
1k views

Analytic continuation of the Dirichlet generating series of the multiplicative partition function

Apologies for the lengthy question, but it seems it's the only way i can convey my thoughts. Consider the Dirichlet series: $$\kappa(s)=\prod_{m=2}^{\infty}\frac{1}{1-m^{-s}}=\sum_{n=1}^{\infty}\frac{\...
mohammad-83's user avatar
13 votes
0 answers
306 views

Upper bound on prime powers in interval

I just spent a full day on the brutish and thankless task of proving that the Brun-Titchmarsh bound holds for prime powers (including primes), and not just for primes, in the following senses: (a) the ...
H A Helfgott's user avatar
  • 19.4k
13 votes
0 answers
453 views

The trace formula over function fields

There are many examples in number theory where an "arithmetic" problem (i.e. for number fields) has an easier analogue for function fields over finite fields. This is also true for questions ...
user125639's user avatar
13 votes
0 answers
611 views

No Siegel-Landau zeros for $\mathrm{GL}(n)$

The problem of non-existance of Siegel-Landau zeros seems to be uncharacteristically easier for cuspidal automorphic representations $\pi$ on $\mathrm{GL}(n)$ if $n\geq2$. We have in fact: There ...
Myshkin's user avatar
  • 17.4k
13 votes
0 answers
1k views

Small primes attract large primes

I posted a version of this to stackexchange and got 12 up-votes and no answers in somewhat more than a day. Someone in a comment construed it as asking for a lot of novel research including figuring ...
Michael Hardy's user avatar
13 votes
0 answers
1k views

A question about Mobius inversion

I don't know how precise I can make this question. I want to know whether there is a theorem that says that a certain phenomenon always happens, but I think the best I can do in order to pin down the ...
gowers's user avatar
  • 28.7k
13 votes
0 answers
498 views

Making a character small at a reciprocal

The following question emerged from thinking about the Erdős discrepancy problem. I don't know whether an answer would be directly helpful, but it might, and in any case I find the question quite ...
gowers's user avatar
  • 28.7k
12 votes
0 answers
375 views

Computing Mertens' function in time O(sqrt(x)) - in practice

As far as I know, there is one way currently known to -- in principle -- compute the Mertens function $M(x) = \sum_{n\leq x} \mu(n)$ in time essentially $O\left(x^{1/2}\right)$, namely, a modification ...
H A Helfgott's user avatar
  • 19.4k
12 votes
0 answers
888 views

Primes and Parity

This problem is motivated by the polymath4 project. There, the aim was to find an efficient deterministic algorithm for finding a prime larger than $N$. The hope was to find a polynomial algorithm in $...
Gil Kalai's user avatar
  • 24.2k
12 votes
0 answers
484 views

Weyl law for Maass forms with nontrivial character

The classical Weyl law for $\Gamma = \mathrm{SL}_2(\mathbb{Z})$ counts the number of Maass cusp forms on $\Gamma \backslash \mathbb{H}$ with Laplace eigenvalue less than $T$. This is originally due to ...
Peter Humphries's user avatar
12 votes
0 answers
617 views

Sieve bound for prime $k$-tuples

Let $d_1<d_2<\dots<d_k$ be integers. Then the number of integers $n\leq x$, such that $n+d_1, n+d_2, \ldots, n+d_k$ are simultaneously prime, is bounded above by $$ \mathfrak{S}(d_1, \ldots, ...
Jan-Christoph Schlage-Puchta's user avatar
11 votes
0 answers
236 views

Sums of $\Lambda(n)$ or $\mu(n)$ with hyperbolic-function weights: surprise!

I was leafing through Gradshteyn–Ryzhik in bed yesterday, as one does, and noticed on the last page that the Mellin transforms of several hyperbolic functions have a factor of $\zeta(s-1)$ or $\zeta(s)...
H A Helfgott's user avatar
  • 19.4k
11 votes
0 answers
424 views

Growth of $n=n(k)$ for which there's a non-trivial solution to $x_1^k+\cdots+x_n^k=y^k$?

Walter Hayman just asked me the following question. What, if anything, is known about the growth of the function $n(k)$, where $k\geq1$ is an integer, and $n=n(k)\geq2$ is the smallest integer for ...
Kevin Buzzard's user avatar
10 votes
0 answers
411 views

Are prime numbers among sums of prime numbers distributed as $\frac n{2\ln(n)}$?

Let $(s_n)_{n\in\mathbb N}$ be defined as follows: For $n\in\mathbb N$, $s_n:=2+3+5+\cdots+p_n$ is the sum of the first $n$ prime numbers (e.g.: $s_1=2$, $s_2=5$, $s_3=10$, $s_4=17$, $\ldots$). Let $\...
Tobias Schnieders's user avatar
10 votes
0 answers
245 views

A weighted count of Egyptian fraction representations

Previously asked and bountied at MSE: Given a positive rational $q$, let $$\mathsf{E}(q)=\left\{X\in[\mathbb{N}]^{\mathit{fin}}: q=\sum_{x\in X}{1\over x}\right\}$$ be the set of Egyptian fraction ...
Noah Schweber's user avatar
10 votes
0 answers
322 views

Are there are any attempts utilising sieve theory to attack the general $a p \pm 1$ problem?

It is currently an open question if there are infinitely many primes $p$ such that $2p + 1$ is prime (Sophie Germain primes) or that at least one of $24p \pm 1$ is prime. Could Zhang's method, or the ...
KStarGamer's user avatar
10 votes
0 answers
321 views

What happened to Stark's book on the analytic theory of algebraic numbers?

I just read the excellent chapter 6 Galois Theory, Algebraic Numbers and Zeta Functions(*) in Waldschmidt, Michel, et al., eds. From number theory to physics. Berlin etc.: Springer, 1992 by Harold ...
A. Bailleul's user avatar
  • 1,174
10 votes
0 answers
535 views

Bounding $1/\zeta(s)$ given RH

Let $T\geq 0$. Assume RH(T+100), that is, assume that all non-trivial zeros $\rho$ of the Riemann zeta function with $|\Im(\rho)|\leq T+100$ satisfy $\Re(\rho)=1/2$. Can we then give a good upper ...
H A Helfgott's user avatar
  • 19.4k
10 votes
0 answers
263 views

On the infinity of $\{p\in \mathbb {N}:\exists n\in\mathbb{N}~p| \left \lfloor{r^n}\right \rfloor\}$

I've already asked this same question on MSE here, but didn't get much help, so I will try on this site as well. For which $r\in\mathbb{R}$ is the set $\mathscr{P}_r=\{p \in \mathbb{P}:\ (\exists n\...
Lucio Tanzini's user avatar
10 votes
0 answers
225 views

Strong uniqueness of Euler's totient function

Let $f:\mathbb N\to \mathbb C$ be some arithmetical function. Define $\varphi_f(n)$ by the following formula: $$ \varphi_f(n)=\sum_{\substack{k\leq n \\ (k,n)=1}}f(k). $$ In other words, $\varphi_f(...
Alexander Kalmynin's user avatar
10 votes
0 answers
252 views

How many partition values are expected to be prime?

Let $p(n)$ be the partition function. Let $P(N)$ count how many $1\leq n\leq N$ are such that $p(n)$ is prime. Are there any heuristics for how $P(N)$ should behave? A crude guess at how this ...
Thomas Bloom's user avatar
  • 6,608
10 votes
0 answers
2k views

Questions on de Branges' work on the Riemann hypothesis

According to Wikipedia, Louis de Branges de Bourcia has obtained some notable results, such as a proof of the Bieberbach conjecture in 1985, which is now known as de Branges' theorem. Initially, his ...
mayank's user avatar
  • 163
10 votes
0 answers
730 views

Implications of divergence of $1/\zeta(s) $ at 1/2

$1/\zeta(s)=\sum_{n>0}\frac{\mu(n)}{n^s}$ where $\mu$ is the Moebius function. This series is known to converge for $s\ge 1$ and diverge for $s\le 1/2$. Its convergence is unknown if $1/2< s&...
Koushik's user avatar
  • 2,076
10 votes
0 answers
506 views

Montgomery's conjecture and lower bound on certain Fourier transform.

Recently I have come across the following question, while meditating about Matt Young's answer to this question of mine, explaining the heuristic (or at least, one possible heuristic) behind ...
Joël's user avatar
  • 25.7k
10 votes
0 answers
436 views

Evaluating Shintani cone zeta functions

Hi everyone I am trying the evaluate sums of the form $$ \sum_{n_1>0,n_2>0,\ldots,n_m>0} \frac{1}{\big((a_{1,1}n_1 +\ldots +a_{1,m}n_m)^k \ldots (a_{m,1}n_1+ \ldots +a_{m,m}n_m)^k\big)}$$ ...
user3628's user avatar
  • 265
10 votes
1 answer
445 views

On random divisor sums modulo $2^k$

Let $k,n,\ell$ be positive integers with $k,n\ge 2$ and $0\le \ell \le k-1$. For each integer $2\le j \le n$, choose a divisor $d_j$ of $j$, uniformly at random from the divisors of $j$. We denote by $...
Sameer Kailasa's user avatar
9 votes
0 answers
304 views

Best smoothing for the Prime Number Theorem?

There are plenty of proofs of the Prime Number Theorem with explicit error terms - it actually looks like a rather competitive field (see Remark 1.4 in https://arxiv.org/pdf/2204.02588.pdf). Several ...
H A Helfgott's user avatar
  • 19.4k
9 votes
0 answers
412 views

From holes in the image of peculiar functions to new perspective on the Riemann Hypothesis

I am working with the Dirichlet eta function $\eta(z)$, with $z=\sigma+it$, $\sigma > \frac{1}{2}$, and $t>0$. Let us define $$\eta_n(z,\gamma)= \sum_{k=1}^n (-1)^{k+1}\lambda_k^{-\sigma} e^{-it\...
Vincent Granville's user avatar
9 votes
0 answers
198 views

Unexpected patterns on the graph of an L-function on the critical line

Let $L(s)$ be the $L$-function associated to the (only) classical modular form of weight $26$ and level $1$. The completed L-function $\Lambda(s)=2(2\pi)^{-s}\Gamma(s) L(s)$ is symmetric with respect ...
LeechLattice's user avatar
  • 9,421
9 votes
0 answers
242 views

Exponential sums over integers with a fixed number of prime divisors

Are there bounds in the literature on sums of the form $$\sum_{\omega(n)= k} e(\alpha n) \;\;\;\;\;\text{or}\;\;\;\;\; \sum_{\Omega(n)=k} e(\alpha n)$$ for $\alpha$ on minor arcs (i.e., not very close ...
H A Helfgott's user avatar
  • 19.4k
9 votes
0 answers
222 views

Explicit bound for sum of Kloosterman sums

What are the best fully explicit upper bounds one can give for the sum $$\left\lvert \sum_{n=N}^{\infty} \frac{S(a,b;n)}{n} \,I_1\!\left(\frac{4 \pi \sqrt{|ab|}}{n}\right) \right\lvert$$ where $S(a,b;...
Fredrik Johansson's user avatar
9 votes
0 answers
254 views

How small may the discriminant of an $S_d$-field be?

In every degree $d$, the Galois closure of the typical number field has the maximal possible Galois group $S_d$. Denote by $f(d)$ the least absolute value of a discriminant of an $S_d$-field of degree ...
Vesselin Dimitrov's user avatar
9 votes
0 answers
378 views

Number fields ordered by discriminant

Since the discriminant of a number field $K \neq \mathbb{Q}$ is bounded from below by an exponential of the degree $[K:\mathbb{Q}]$, for instance by Minkowski's Geometry of Numbers bound, there are ...
Vesselin Dimitrov's user avatar
9 votes
0 answers
244 views

Semi-primes represented by quadratic polynomials

According to Lemke-Oliver, irreducible quadratic polynomials $G$ with positive leading coefficient and $\rho(2)<2$, (where $\rho(m)$ denotes the number of incongruent solutions to the congruence $G(...
Delmastro's user avatar
  • 195
9 votes
0 answers
411 views

Number of prime factors in a very short interval

Let $k \geq 3$ be a (large enough) integer, let $x \in \mathbb{R}$, and set $I_x := [x, x + \log^k x]$. Some believe that for $x$ large enough there exists a prime $n \in I_x$. Equivalently, there ...
Pablo's user avatar
  • 11.2k
9 votes
0 answers
269 views

Cancellation in a sum of Möbius evaluated along a quadratic form

Let $Q(x,y)$ be an indefinite binary quadratic form. Suppose $0 < B < \sqrt{A} $ are such that $B \gg \sqrt{A}$. Is it true one can save an arbitrary power of log from the trivial bound in $$...
George Shakan's user avatar

1
2 3 4 5
19